3 Stars and 5 Hearts is worth 27 points. 5 Stars and 7 Hearts is worth 41 points. How many points is 1 Star and 1 Heart worth?

Answers

Answer 1

Answer:

1 star = 4 points

1 heart = 3 points

Step-by-step explanation:

3 stars (each star is 4 points) = 3 x 4 = 12

5 hearts (each heart is 3 points) = 5 x 3 = 15

12 + 15 = 27

This applies to the other eaxmple as well.


Related Questions

Solve the system by substitution.
y = -3x + 17
2x + 3y = -5

Answers

Answer:

(8, -7)

Step-by-step explanation:

Hi there!

We are given this system:

y=-3x+17

2x+3y=5

And we want to solve the system

Let's solve it by substitution.

In the first equation, we are given y=-3x+17, which we can substitute -3x+17 as y into the second equation

2x+3(-3x+17)=5

Now do the distributive property

2x+-9x+51=-5

Combine like terms

-7x+51=-5

Subtract 51 from both sides

-7x=-56

Divide both sides by -7

x=8

Now substitute 8 as the value of x in y=-3x+17 to find the value of y

y=-3(8)+17

y=-24+17

y=-7

The answer is x=8, y=-7. If you need it as a point, it's (8, -7)

Hope this helps!

17. Find an equation of the straight line that is perpendicular to x - y = 5 and passes through the point (8, -1).

18. Find the equation of the line in slope-intercept form that passes through the points (12, 4) and (4, 6)

Answers

Answer:

17.y=-x+12

18.y=-1/4x+7

Step-by-step explanation:

17.a Solve for y (y=-5+x)

17. b Reciprocal (y=-1x-5)

17. c Subsitute equation into (8,-1) (-1=-13=y=x+12)

18.a Subtract y values and x values= (6-4)/(4-12)=2/-8=-1/4, this will be your slope

18. b Plug in -1/4 into either of the two ordered pairs to find y value=(12,4)=(4=-3)=y value=7.

18. c=y=-1/4x+7

Find the third angle of a triangle if two of its angles are equal to:

25 degrees and 80 degrees

and

25 degrees and 120 degrees

Answers

Answer:

60 degrees

Step-by-step explanation:

Answer:

75 degrees, 35 degrees

Step-by-step explanation:

a. Adding 25 degrees and 80 degrees yields 105 degrees.  The third angle of the given triangle must therefore be 180 degrees less 105 degrees, or 75 degrees.

b. Adding 25 degrees and 120 degrees yields 145 degrees.  The third angle of the given triangle must therefore be 180 degrees less 145 degrees, or 35 degrees.

Find the slope of the line that passes through the following points: (0, -3), (-6,8) ​

Answers

Answer:

-11/6

Step-by-step explanation:

To find the slope, we use the formula

m = ( y2-y1)/(x2-x1)

   = ( 8- -3)/( -6 - 0)

   = (8+3)/(-6-0)

  = 11 / -6

 = -11/6

Need help ASAP!!!Please explain how to solve the problem

Answers

Answer:

Step-by-step explanation:

(b) Work out the probability Jennifer hits the Bullseye at least once.


Can someone work this out for me ?

Answers

Answer:

1/2

Step-by-step explanation:

Filling out the tree diagram,

On the first throw, there is a 4/4 = 100%* chance Jennifer hits or misses. Therefore, the chance she misses is equal to the difference between

(chance she hits or misses) - (chance she hits) = 4/4 - 1/4 = 3/4

For the second throw, the chance she hits is independent of whether she hits or misses on the first throw. This means that it does not matter whether she hits or misses on the first throw; her chance to hit the second throw will always be 1/3. Therefore, 1/3 can go into the hit category on each hit line on the second throw, and in the miss category, we can put

100% - 1/3 = 1 - 1/3 = 3/3 - 1/3 = 2/3

For Jennifer to hit the bullseye at least once, she must either:

- Hit and then hit

- Hit and then miss

- Miss and then hit

From this, we can determine that if Jennifer hits the first bullseye, she will have hit it at least once, no matter what. Therefore, Jennifer must either:

- Hit the first one

- Miss and then hit

In probability, OR /either means that we add the probabilties. The probability that she hits the first one is 1/4, and the probability that she misses and then hits is

(probability she misses the 1st one) * (probability she hits the second one). We know to multiply here because both scenarios must happen, and for AND probabilities, we multiply. We thus have

(3/4) * (1/3) = 3/12 = 1/4 as the probabilty that she misses then hits, and 1/4 as the probability that she hits the first one. We add these up to get 1/4 + 1/4 = 2/4 = 1/2 as our probability that she hits the bullseye at least once

* we know 4/4 = 100% because 100% = 1, and anything divided by itself is equal to 1.

How many more unit tiles must be added to the function f(x)=x2−6x+1 in order to complete the square? 1 6 8 9

Answers

Answer:

easy

Step-by-step explanation:

8

Answer:

its 8

Step-by-step explanation:

Find the measure of the numbered angles.
Angle 1 =
Angle 2 =
Angle 3 =
Angle 4 =
Angle 5 =
Angle 6 =
Angle 7 =
Angle 8 =

Answers

Explanation in pictures.

Hope this helps!! Have a good day!

Help please anyoneee

Answers

Answer:

here's the answer to your question

the angle of elevation to a nearby tree from a point of the ground is measured to be 39 degrees

Answers

Answer:

66.4

Step-by-step explanation:

tan(39)=x/82

x=tan(39)×82

x=66.4 (rounded to the nearest tenth)

Answered by GAUTHMATH

I need help with this does anybody know the answer

Answers

Answer:

X = 50; Y = 50

Answers are the same by following the correct order of operations.

Step-by-step explanation:

This problem deals with the correct order of operations. Multiplication is done before addition.

22 * 2 + 6 = X

44 + 6 = X

X = 50

6 + 22 * 2 = Y

6 + 44 = Y

Y = 50

The answers are the same despite the order in which the operations were written since by following the correct order of operations, you must do the multiplication first in each chase.

Which expression is equivalent to d - 8 - 8d?



Answers

Answer:

−7d−8

Step-by-step explanation:

Subtract 8d from d.

form a polynomial whose factors are (x+1) , (x+2) and (x-3)​

Answers

Answer:

x³-7x-6

Step-by-step explanation:

Find the products of the factors

(x+1)(x+2)(x-3)

x²+3x+2(x-3)

x³+3x²+2x-3x²-9x-6

x³-7x-6

Joylin is writing an equation to model the proportional relationship between y, the total cost in dollars of downloading videos from a website, and x, the number of videos downloaded. She knows that the total cost to download 3 videos was $12. Her work to find the equation is shown below.

Joylin’s Work

Step 1
k = StartFraction 3 over 12 EndFraction = 0.25
Step 2
y = 0.25 x

Where did Joylin make her first error?

Answers

Answer:

Step-by-step explanation:

Jocelyn was attempting to find out how much she will be charged per download. She has the fraction upside down. To find the amount of money per download, the fraction should be $12/3downloads to get $4/1download. That is the slope of the equation, the rate of change or, for us, the fact that your cost will go up $4 for every single video you download. The equation would then be

y = 4x

The way she has it, we are paying only a quarter for a download. If that be the case, for 3 downloads we would only be paying .75, but it says we pay $12, so we know something is wrong right there.

Answer:

The answer is A

Step-by-step explanation:

please help, i will give brainliest!

Answers

Answer:

yes please, ask the question

3

Select the correct answer.

The angle of depression between the top of a 100-foot cliff and a ship approaching the shore is 37°.

cliff top

37°

100

feet

37°

ship

d

What is the approximate distance, d, between the bottom of the cliff and the ship?

ОА

166.2 feet

OB. 60.2 feet

ОС.

75.4 feet

OD.

132.7 feet

Reset

Next

Answers

Answer:

132.7 feet.

Step-by-step explanation:

tan 37 = height of cliff / distance of the ship

=  100/d

d = 100/tan37

= 132.7 feet.

HELP PLS!! ILL GIVE POINTS !! :(( m

Find the product of the complex numbers. Express your answer in
trigonometric form.
Z1 =5(cos15° + i sin 15°)
Z2=3(cos70° + i sin 70°)
A. 8(cos85° + isin85°)
B. 2 (cos(-55°) + i sin(-55°))
C. 2 (cos305° + i sin 305°)
D. 15(cos85° + i sin85°)

Answers

Answer:  Choice D

15(cos85° + i sin85°)

===========================================================

Explanation:

Let's say we had these two general complex numbers, which are in polar form.

[tex]z_1 = r_1*\left(\cos(\theta_1)+i*\sin(\theta_1)\right)\\\\z_2 = r_2*\left(\cos(\theta_2)+i*\sin(\theta_2)\right)\\\\[/tex]

We can abbreviate them into the shorthand form

[tex]z_1 = r_1*\text{cis}(\theta_1)\\\\z_2 = r_2*\text{cis}(\theta_2)\\\\[/tex]

The notation "cis" stands for "cosine i sine".

Now that we have those complex numbers set up, multiplying them is as simple as saying this:

[tex]z_1*z_2 = (r_1*r_2)*\text{cis}(\theta_1+\theta_2)[/tex]

We do two basic things:

Multiply the r values out frontAdd the theta values inside the the cis function

---------------------------------------

With all that in mind, let's tackle the problem your teacher gave you.

The given complex numbers

[tex]z_1 = 5*\left(\cos(15^{\circ})+i*\sin(15^{\circ})\right)\\\\z_2 = 3*\left(\cos(70^{\circ})+i*\sin(70^{\circ})\right)\\\\[/tex]

abbreviate into

[tex]z_1 = 5*\text{cis}(15^{\circ})\\\\z_2 = 3*\text{cis}(70^{\circ})\\\\[/tex]

then those multiply to

[tex]z_1*z_2 = (r_1*r_2)*\text{cis}(\theta_1+\theta_2)\\\\z_1*z_2 = (5*3)*\text{cis}(15+70)\\\\z_1*z_2 = 15\text{cis}(85^{\circ})\\\\z_1*z_2 = 15\left(\cos(85^{\circ})+i\sin(85^{\circ})\right)\\\\[/tex]

which is why choice D is the final answer.

An athlete throws her javelin 9.5 yards. She throws it this distance 2 times. How far she throw her javelin in total

Answers

Answer: 19 yards

Step-by-step explanation: 9.5x2=19

Translate to an equation, then solve.
Nineteen less than six times a number is equal to 11 more than four times the number?
What is the number?​

Answers

Answer:

15

Step-by-step explanation:

Let the number is x, then we have:

6x - 19 = 4x + 116x - 4x = 11 + 192x = 30x = 15

(4,-5) with slope of 2

Answers

that's a point slope form so

y +5 = 2(x -4)

x.(9x-1).(x+2)-x(3x-1).(3x+1)

Answers

Answer:

Sorry if i'm wrong.

=6x² + 16x - 2

Which shows two triangles that are congruent by ASA?

Answers

Answer:

A and D

Step-by-step explanation:

The two triangles that are congruent by ASA is discussed below.

What is Congruency?

Two figures are said to be "congruent" if they can be positioned perfectly over one another. Both of the bread slices are the same size and shape when stacked one on top of the other. "Congruent" refers to things that are precisely the same size and shape.

Angle-Side-Angle refers to the ASA Congruence rule.

According to this rule, two triangles are said to be congruent if any two of their respective angles and the side that is included between them are equal to the corresponding angles and the included side of the other triangle.

For example, there are ABC and DEF in which ∠ B = ∠ E, ∠ C = ∠ F, and BC = EF

Then, the triangle that Δ ABC ≅ Δ DEF by ASA congruence rule.

Learn more about Congruency here:

https://brainly.com/question/7888063

#SPJ5

Two friends go to a Mexican restaurant for lunch. One friend orders 1 soft taco and
3 burritos and his bill was $8.75. The other friend's bill was $10.00 and he ordered 4
soft tacos and 2 burritos. What is the cost of a taco and what is the cost of a
burrito?
The cost of a taco is:
The cost of a burrito is:

Answers

Answer:

cost of a taco: $1.25

cost of a burrito: $2.50

Step-by-step explanation:

Let the cost of a taco be $t and the cost of a burrito be $b.

t +3b= 8.75 -----(1)

4t +2b= 10

Divide both sides by 2:

2t +b= 5

Making b the subject of formula:

b= 5 -2t -----(2)

Substitute (2) into (1):

t +3(5 -2t)= 8.75

Expand:

t +3(5) +3(-2t)= 8.75

t +15 -6t= 8.75

t -6t= 8.75 -15

-5t= -6.25

t= -6.25 ÷(-5)

t= 1.25

Substitute t= 1.25 into (2):

b= 5 -2(1.25)

b= 5 -2.5

b= 2.50

Thus, the cost of a taco is $1.25 and the cost of a burrito is $2.50.

PLEASE HELP WILL GIVE BRAINLIEST!!!!

Answers

Answer:

wqhg

Step-by-step explanation:

its situated in u.s, in quahog , rhode island

You are taking a survey on the heights of all your classmates. This is an example of categorical data.
True or false?

Answers

Answer:

trueeeeeeeeeeeeeeeeeee

13 Water is pulled up from a well in a bucket on a rope. The rope winds on a cylindrical drum 15 cm in diameter. It takes 28 turns of the drum to pull the bucket up from the bottom of the well. How deep is the well? (Use the value 2 for m.)​

Answers

Multiply 15 by 28 and then times it by 13??

Find f(-3) if f(x) = x2 .


Type a numerical answer in the space provided. Do not type spaces in your answer.

Answers

Answer:

f(-3) = 9

General Formulas and Concepts:

Pre-Algebra

Order of Operations: BPEMDAS

Brackets Parenthesis Exponents Multiplication Division Addition Subtraction Left to Right

Algebra I

Functions

Function Notation

Step-by-step explanation:

Step 1: Define

Identify

f(x) = x²

Step 2: Evaluate

Substitute in x [Function f(x)]:                                                                          f(-3) = (-3)²Exponents:                                                                                                        f(-3) = 9

PLZZZ HELPPP, IF NOT 100% SURE PLZ DONT ANSWER…BRAINLIEST TO FIRST AND CORRECT ANSWER, THX TO SECOND AND CORRECT ANSWER

Answers

Answer:  4

=============================================================

Explanation:

1/2 = 4/8 after multiplying top and bottom by 8

So the mixed number 7  1/2 is the same as 7  4/8

We can convert to an improper fraction like so

7  4/8

7 + 4/8

7*(8/8) + 4/8

56/8 + 4/8

(56+4)/8

60/8

Notice how dividing 60 over 8 leads to 7 remainder 4. Think of it like having 60 cookies and you want to share them amongst 8 friends. Each friend gets 7 whole cookies (7*8 = 56 taken so far) and then we have 60-56 = 4 left over as the remainder.

Through similar steps, you should find that the mixed number 1  7/8 converts to the improper fraction 15/8.

-----------------------------

To rephrase the problem with improper fractions would be to say:

"The electrician needs to buy 60/8 feet of wire. There are 15/8 feet of wire in each spool. How many spools should the electrician buy?"

Let x be the answer to that question.

We multiply the number of spools (x) by the amount of feet of wire per spool (15/8) to get the expression (15/8)x

Set this equal to the target 60/8 and solve for x

(15/8)x = 60/8

15x = 60 .... multiply both sides by 8 to clear out the fractions

x = 60/15

x = 4

The electrician needs to buy exactly 4 spools

-----------------------------

A different approach could have us convert each mixed number into decimal form

7  1/2 = 7 + 1/2 = 7 + 0.5 = 7.5

1  7/8 = 1 + 7/8 = 1 + 0.875 = 1.875

So the electrician needs to buy 7.5 ft total and each spool has 1.875 ft of wire.

Using the same idea as before, we would then have,

1.875x = 7.5

x = (7.5)/(1.875)

x = 4

Write the function for the graph.

Answers

Answer:

B)

Step-by-step explanation:

exponential function given by:

[tex] \displaystyle f(x )= a \cdot{b}^{x} [/tex]

where:

a is the y-interceptb can be any number

since the function crosses y-axis at (0,4) point

[tex]a = 4[/tex]

to find a we can consider (1,8) point it tells us when x is 1 then y is 8 thus:

[tex] \displaystyle 8= 4 \cdot{b}^{1} [/tex]

[tex] \displaystyle 8= 4 \cdot{b}^{} [/tex]

divide both sides by 4:

[tex] \displaystyle \boxed{b = 2}[/tex]

altogether substitute:

[tex] \displaystyle f(x)= 4 \cdot{2}^{x} [/tex]

hence our answer is B)

please help. people take my points without answering. 15 points left, please show work. and please dont just take my points

Answers

Answer:

Step-by-step explanation:

Reference angle in quadrant I,

θ' = θ

Reference angle in quadrant II,

θ' = 180° - θ

Reference angle in quadrant III,

θ' = θ - 180°

Reference angle in quadrant IV,

θ' = 360° - θ

13). θ = 315°

Since, the given angle is in the 4th quadrant,

Reference angle θ' = 360° - 315°

                                 = 45°

14). θ' = (620° - 540°)

         = 80°

Reference angle = 80°

15). θ' = 630° - θ

          = 630° - 580°

          = 50°

Reference angle = 50°

Other Questions
ASAP PLZ I NEED IT RNWho organized the District 21 branch of the United Mine Workers? A. Pete Hanraty B. J. J. McAlester C. H. W. Faucett D. Lewis Ross 4 ptsJohn makes $2,800 per month and has an opportunity to invest $150 per month at an APR of 4.5% in a 401K plan through work. He plans to retire in 30 years.Type in the EXACT formula you would use in excel (no spaces) to find the amount in the account after he has invested for 30 years.Using excel, what is the answer to the formula from the previous part?How much money will he deposit into the 401K over the 30 years?How much total interest will he earn on his 401K over the 30 years? Why do you think its a challenge to reduce fossil fuel usage in homes in the United States? How could the use of fossil fuels affect the biodiversity in neighborhoods and the surrounding ecosystems? Use a table of values to graph the function (x) = x. Choose the correct graph from the options below. Communication style, education, family status, military experience, organization role and level, religion, first language, geographical location, income, work experience, and work style are all considered ______ dimensions of diversity. primary secondary quaternary tertiary What is the volume of this right triangular pyramid?Height -36Base:side 1 -24side 2 -192736 cm5472 cm8208 cm 16,416 cm Choisissez le bon adjectif dmonstratif et le bon suffixe pour complter la traduction.They buy mutton at this butcher shop. - Ils achtent du mouton ________. ces boucherie-l cette boucherie-l ce boucherie-ci cette boucherie-ci Which functional group is used in other functional groups? A. Ester B. Carbonyl c. Hydroxyl D. Amino q9 Find the expression that is equivalent to 7(x2 5x + 1). Place and convenience are connected by a core linkage. While GoPro was able to get the product into locations where customers could find it, it made an error when production problems forced it to what are the differences between pituitary and thyroid dwarfism? All of the following are reasons to participate in a job shadow except:A. seeing the connection of school to work.B. learning from experts in a field.C. getting a close look at various careers.D. having the opportunity to gain school credit.(A-P-E-X)I give Brainliest for the correct answer 123Which expression is equivalent to | 18 ? Assume x 20.o3xO302x3 A company purchased $1,800 of merchandise on July 5 with terms 2/10, n/30. On July 7, it returned $200 worth of merchandise. On July 28, it paid the full amount due. Assuming the company uses a perpetual inventory system, and records purchases using the gross method, the correct journal entry to record the merchandise return on July 7 is: On June 30, 2021, Moran Corporation issued $4 million of its 8% bonds for $3.5 million. The bonds were priced to yield 9.4%. The bonds are dated June 30, 2021. Interest is payable semiannually on December 31 and July 1. If the effective interest method is used, by how much should the bond discount be reduced for the six months ended December 31, 2021? a) $3,500. b) $4,500. c) $4,800. d) $9,000. Find the perimeter, in inches, of the parallelogram.A parallelogram is 5 inches wide, 15 inches long, and 4 inches high. which is the best paraphrase of the following quote? please put away all of your books and materials before leaving today, class remarked mrs leon.A.) Mrs. Leon asked the class to put away all of their books and materials before going to lunch.B.) the class told Mrs. Leon they would put away all of their books and materials before leaving C.) The class told Mrs. Leon they would put away all of their book and materials before going to lunch .D.) Mrs. Leon asked the class to put away all of their books and materials before leaving. A court decision that was applied to a previous case that is used to guide how to decide on a current issue is known as c. Assuming a fixed-priority scheduling. Consider two tasks to be executed periodically on a single processor, where task 1 has period p= 4 and task 2 has period p= 6. If execution time of task I be e;=1 Find the maximum value for the execution time e2 of task 2 such that the Rate Monotonic (RM) schedule is feasible. Donde esta juan? Juan fue a la tienda